Is this set perfect?












0












$begingroup$


Let $X$ be a compact and Hausdorff space and $f:Xto X$ is a homeomorphism.
Let for all $ineq j$ and $pin X$, we have $f^i(p)neq f^j(p)$. I need to know that whether $overline{{f^n(p)}_{ninmathbb{Z}}}$ is uncountable or not. I think that it is sufficient to $overline{{f^{n}(p)}_{ninmathbb{Z}}}$ is an infinite and closed without isolated point i.e. it is a perfect set. It is clear that it is infinite and close. Let $p$ be an isolated point, this implies that there is an open set $U$ such that $U={p}$, hence ${f^n(U)}_{ninmathbb{Z}}$ is an open cover for compact set $overline{{f^{n}(p)}_{ninmathbb{Z}}}$. Thus there is $m>0$ with $overline{{f^{n}(p)}_{ninmathbb{Z}}}subseteq cup_{n=0}^{m}f^n(U)$, that is a contradiction.



Is my proof true?
Please help me










share|cite|improve this question











$endgroup$

















    0












    $begingroup$


    Let $X$ be a compact and Hausdorff space and $f:Xto X$ is a homeomorphism.
    Let for all $ineq j$ and $pin X$, we have $f^i(p)neq f^j(p)$. I need to know that whether $overline{{f^n(p)}_{ninmathbb{Z}}}$ is uncountable or not. I think that it is sufficient to $overline{{f^{n}(p)}_{ninmathbb{Z}}}$ is an infinite and closed without isolated point i.e. it is a perfect set. It is clear that it is infinite and close. Let $p$ be an isolated point, this implies that there is an open set $U$ such that $U={p}$, hence ${f^n(U)}_{ninmathbb{Z}}$ is an open cover for compact set $overline{{f^{n}(p)}_{ninmathbb{Z}}}$. Thus there is $m>0$ with $overline{{f^{n}(p)}_{ninmathbb{Z}}}subseteq cup_{n=0}^{m}f^n(U)$, that is a contradiction.



    Is my proof true?
    Please help me










    share|cite|improve this question











    $endgroup$















      0












      0








      0





      $begingroup$


      Let $X$ be a compact and Hausdorff space and $f:Xto X$ is a homeomorphism.
      Let for all $ineq j$ and $pin X$, we have $f^i(p)neq f^j(p)$. I need to know that whether $overline{{f^n(p)}_{ninmathbb{Z}}}$ is uncountable or not. I think that it is sufficient to $overline{{f^{n}(p)}_{ninmathbb{Z}}}$ is an infinite and closed without isolated point i.e. it is a perfect set. It is clear that it is infinite and close. Let $p$ be an isolated point, this implies that there is an open set $U$ such that $U={p}$, hence ${f^n(U)}_{ninmathbb{Z}}$ is an open cover for compact set $overline{{f^{n}(p)}_{ninmathbb{Z}}}$. Thus there is $m>0$ with $overline{{f^{n}(p)}_{ninmathbb{Z}}}subseteq cup_{n=0}^{m}f^n(U)$, that is a contradiction.



      Is my proof true?
      Please help me










      share|cite|improve this question











      $endgroup$




      Let $X$ be a compact and Hausdorff space and $f:Xto X$ is a homeomorphism.
      Let for all $ineq j$ and $pin X$, we have $f^i(p)neq f^j(p)$. I need to know that whether $overline{{f^n(p)}_{ninmathbb{Z}}}$ is uncountable or not. I think that it is sufficient to $overline{{f^{n}(p)}_{ninmathbb{Z}}}$ is an infinite and closed without isolated point i.e. it is a perfect set. It is clear that it is infinite and close. Let $p$ be an isolated point, this implies that there is an open set $U$ such that $U={p}$, hence ${f^n(U)}_{ninmathbb{Z}}$ is an open cover for compact set $overline{{f^{n}(p)}_{ninmathbb{Z}}}$. Thus there is $m>0$ with $overline{{f^{n}(p)}_{ninmathbb{Z}}}subseteq cup_{n=0}^{m}f^n(U)$, that is a contradiction.



      Is my proof true?
      Please help me







      general-topology dynamical-systems






      share|cite|improve this question















      share|cite|improve this question













      share|cite|improve this question




      share|cite|improve this question








      edited Dec 15 '18 at 9:29









      Jean Marie

      30.5k42154




      30.5k42154










      asked Dec 15 '18 at 9:05









      user479859user479859

      887




      887






















          1 Answer
          1






          active

          oldest

          votes


















          0












          $begingroup$

          There are several problems with your proof:




          1. You wrote “Let $p$ be an isolated point”, but $p$ here already has another meaning.

          2. Then you write “this implies that there is an open set $U$ such that $U={p}$”. This is wrong. What it implies is that there is an open subset $U$ of $X$ such that $overline{{f^n(p),|,ninmathbb{Z}}}cap U={p}$.






          share|cite|improve this answer











          $endgroup$













          • $begingroup$
            @ José Carlos Santos. Thanks. I think that $overline{{f^n(p)}_{ninmathbb{Z}}}$ is an uncountable set. In above proof $f$ is a homeomorphism, hence it is an open map and $f(overline{U})= overline{f(U)}$.
            $endgroup$
            – user479859
            Dec 15 '18 at 9:28












          • $begingroup$
            I've deleted my third remark.
            $endgroup$
            – José Carlos Santos
            Dec 15 '18 at 9:30










          • $begingroup$
            Let $overline{{f^n(p)| ninmathbb{Z}}}cap U={p}$.
            $endgroup$
            – user479859
            Dec 15 '18 at 9:31










          • $begingroup$
            If $overline{f^n(p)| ninmathbb{Z}}}cap U={ f^k(p)}$, in above proof we work with $f^k(p)$.
            $endgroup$
            – user479859
            Dec 15 '18 at 9:32












          • $begingroup$
            What the means of "$ overline{{f^n(p)|ninmathbb{Z}}}$ has an isolated point"?
            $endgroup$
            – user479859
            Dec 15 '18 at 9:35













          Your Answer





          StackExchange.ifUsing("editor", function () {
          return StackExchange.using("mathjaxEditing", function () {
          StackExchange.MarkdownEditor.creationCallbacks.add(function (editor, postfix) {
          StackExchange.mathjaxEditing.prepareWmdForMathJax(editor, postfix, [["$", "$"], ["\\(","\\)"]]);
          });
          });
          }, "mathjax-editing");

          StackExchange.ready(function() {
          var channelOptions = {
          tags: "".split(" "),
          id: "69"
          };
          initTagRenderer("".split(" "), "".split(" "), channelOptions);

          StackExchange.using("externalEditor", function() {
          // Have to fire editor after snippets, if snippets enabled
          if (StackExchange.settings.snippets.snippetsEnabled) {
          StackExchange.using("snippets", function() {
          createEditor();
          });
          }
          else {
          createEditor();
          }
          });

          function createEditor() {
          StackExchange.prepareEditor({
          heartbeatType: 'answer',
          autoActivateHeartbeat: false,
          convertImagesToLinks: true,
          noModals: true,
          showLowRepImageUploadWarning: true,
          reputationToPostImages: 10,
          bindNavPrevention: true,
          postfix: "",
          imageUploader: {
          brandingHtml: "Powered by u003ca class="icon-imgur-white" href="https://imgur.com/"u003eu003c/au003e",
          contentPolicyHtml: "User contributions licensed under u003ca href="https://creativecommons.org/licenses/by-sa/3.0/"u003ecc by-sa 3.0 with attribution requiredu003c/au003e u003ca href="https://stackoverflow.com/legal/content-policy"u003e(content policy)u003c/au003e",
          allowUrls: true
          },
          noCode: true, onDemand: true,
          discardSelector: ".discard-answer"
          ,immediatelyShowMarkdownHelp:true
          });


          }
          });














          draft saved

          draft discarded


















          StackExchange.ready(
          function () {
          StackExchange.openid.initPostLogin('.new-post-login', 'https%3a%2f%2fmath.stackexchange.com%2fquestions%2f3040299%2fis-this-set-perfect%23new-answer', 'question_page');
          }
          );

          Post as a guest















          Required, but never shown

























          1 Answer
          1






          active

          oldest

          votes








          1 Answer
          1






          active

          oldest

          votes









          active

          oldest

          votes






          active

          oldest

          votes









          0












          $begingroup$

          There are several problems with your proof:




          1. You wrote “Let $p$ be an isolated point”, but $p$ here already has another meaning.

          2. Then you write “this implies that there is an open set $U$ such that $U={p}$”. This is wrong. What it implies is that there is an open subset $U$ of $X$ such that $overline{{f^n(p),|,ninmathbb{Z}}}cap U={p}$.






          share|cite|improve this answer











          $endgroup$













          • $begingroup$
            @ José Carlos Santos. Thanks. I think that $overline{{f^n(p)}_{ninmathbb{Z}}}$ is an uncountable set. In above proof $f$ is a homeomorphism, hence it is an open map and $f(overline{U})= overline{f(U)}$.
            $endgroup$
            – user479859
            Dec 15 '18 at 9:28












          • $begingroup$
            I've deleted my third remark.
            $endgroup$
            – José Carlos Santos
            Dec 15 '18 at 9:30










          • $begingroup$
            Let $overline{{f^n(p)| ninmathbb{Z}}}cap U={p}$.
            $endgroup$
            – user479859
            Dec 15 '18 at 9:31










          • $begingroup$
            If $overline{f^n(p)| ninmathbb{Z}}}cap U={ f^k(p)}$, in above proof we work with $f^k(p)$.
            $endgroup$
            – user479859
            Dec 15 '18 at 9:32












          • $begingroup$
            What the means of "$ overline{{f^n(p)|ninmathbb{Z}}}$ has an isolated point"?
            $endgroup$
            – user479859
            Dec 15 '18 at 9:35


















          0












          $begingroup$

          There are several problems with your proof:




          1. You wrote “Let $p$ be an isolated point”, but $p$ here already has another meaning.

          2. Then you write “this implies that there is an open set $U$ such that $U={p}$”. This is wrong. What it implies is that there is an open subset $U$ of $X$ such that $overline{{f^n(p),|,ninmathbb{Z}}}cap U={p}$.






          share|cite|improve this answer











          $endgroup$













          • $begingroup$
            @ José Carlos Santos. Thanks. I think that $overline{{f^n(p)}_{ninmathbb{Z}}}$ is an uncountable set. In above proof $f$ is a homeomorphism, hence it is an open map and $f(overline{U})= overline{f(U)}$.
            $endgroup$
            – user479859
            Dec 15 '18 at 9:28












          • $begingroup$
            I've deleted my third remark.
            $endgroup$
            – José Carlos Santos
            Dec 15 '18 at 9:30










          • $begingroup$
            Let $overline{{f^n(p)| ninmathbb{Z}}}cap U={p}$.
            $endgroup$
            – user479859
            Dec 15 '18 at 9:31










          • $begingroup$
            If $overline{f^n(p)| ninmathbb{Z}}}cap U={ f^k(p)}$, in above proof we work with $f^k(p)$.
            $endgroup$
            – user479859
            Dec 15 '18 at 9:32












          • $begingroup$
            What the means of "$ overline{{f^n(p)|ninmathbb{Z}}}$ has an isolated point"?
            $endgroup$
            – user479859
            Dec 15 '18 at 9:35
















          0












          0








          0





          $begingroup$

          There are several problems with your proof:




          1. You wrote “Let $p$ be an isolated point”, but $p$ here already has another meaning.

          2. Then you write “this implies that there is an open set $U$ such that $U={p}$”. This is wrong. What it implies is that there is an open subset $U$ of $X$ such that $overline{{f^n(p),|,ninmathbb{Z}}}cap U={p}$.






          share|cite|improve this answer











          $endgroup$



          There are several problems with your proof:




          1. You wrote “Let $p$ be an isolated point”, but $p$ here already has another meaning.

          2. Then you write “this implies that there is an open set $U$ such that $U={p}$”. This is wrong. What it implies is that there is an open subset $U$ of $X$ such that $overline{{f^n(p),|,ninmathbb{Z}}}cap U={p}$.







          share|cite|improve this answer














          share|cite|improve this answer



          share|cite|improve this answer








          edited Dec 15 '18 at 9:30

























          answered Dec 15 '18 at 9:20









          José Carlos SantosJosé Carlos Santos

          165k22132235




          165k22132235












          • $begingroup$
            @ José Carlos Santos. Thanks. I think that $overline{{f^n(p)}_{ninmathbb{Z}}}$ is an uncountable set. In above proof $f$ is a homeomorphism, hence it is an open map and $f(overline{U})= overline{f(U)}$.
            $endgroup$
            – user479859
            Dec 15 '18 at 9:28












          • $begingroup$
            I've deleted my third remark.
            $endgroup$
            – José Carlos Santos
            Dec 15 '18 at 9:30










          • $begingroup$
            Let $overline{{f^n(p)| ninmathbb{Z}}}cap U={p}$.
            $endgroup$
            – user479859
            Dec 15 '18 at 9:31










          • $begingroup$
            If $overline{f^n(p)| ninmathbb{Z}}}cap U={ f^k(p)}$, in above proof we work with $f^k(p)$.
            $endgroup$
            – user479859
            Dec 15 '18 at 9:32












          • $begingroup$
            What the means of "$ overline{{f^n(p)|ninmathbb{Z}}}$ has an isolated point"?
            $endgroup$
            – user479859
            Dec 15 '18 at 9:35




















          • $begingroup$
            @ José Carlos Santos. Thanks. I think that $overline{{f^n(p)}_{ninmathbb{Z}}}$ is an uncountable set. In above proof $f$ is a homeomorphism, hence it is an open map and $f(overline{U})= overline{f(U)}$.
            $endgroup$
            – user479859
            Dec 15 '18 at 9:28












          • $begingroup$
            I've deleted my third remark.
            $endgroup$
            – José Carlos Santos
            Dec 15 '18 at 9:30










          • $begingroup$
            Let $overline{{f^n(p)| ninmathbb{Z}}}cap U={p}$.
            $endgroup$
            – user479859
            Dec 15 '18 at 9:31










          • $begingroup$
            If $overline{f^n(p)| ninmathbb{Z}}}cap U={ f^k(p)}$, in above proof we work with $f^k(p)$.
            $endgroup$
            – user479859
            Dec 15 '18 at 9:32












          • $begingroup$
            What the means of "$ overline{{f^n(p)|ninmathbb{Z}}}$ has an isolated point"?
            $endgroup$
            – user479859
            Dec 15 '18 at 9:35


















          $begingroup$
          @ José Carlos Santos. Thanks. I think that $overline{{f^n(p)}_{ninmathbb{Z}}}$ is an uncountable set. In above proof $f$ is a homeomorphism, hence it is an open map and $f(overline{U})= overline{f(U)}$.
          $endgroup$
          – user479859
          Dec 15 '18 at 9:28






          $begingroup$
          @ José Carlos Santos. Thanks. I think that $overline{{f^n(p)}_{ninmathbb{Z}}}$ is an uncountable set. In above proof $f$ is a homeomorphism, hence it is an open map and $f(overline{U})= overline{f(U)}$.
          $endgroup$
          – user479859
          Dec 15 '18 at 9:28














          $begingroup$
          I've deleted my third remark.
          $endgroup$
          – José Carlos Santos
          Dec 15 '18 at 9:30




          $begingroup$
          I've deleted my third remark.
          $endgroup$
          – José Carlos Santos
          Dec 15 '18 at 9:30












          $begingroup$
          Let $overline{{f^n(p)| ninmathbb{Z}}}cap U={p}$.
          $endgroup$
          – user479859
          Dec 15 '18 at 9:31




          $begingroup$
          Let $overline{{f^n(p)| ninmathbb{Z}}}cap U={p}$.
          $endgroup$
          – user479859
          Dec 15 '18 at 9:31












          $begingroup$
          If $overline{f^n(p)| ninmathbb{Z}}}cap U={ f^k(p)}$, in above proof we work with $f^k(p)$.
          $endgroup$
          – user479859
          Dec 15 '18 at 9:32






          $begingroup$
          If $overline{f^n(p)| ninmathbb{Z}}}cap U={ f^k(p)}$, in above proof we work with $f^k(p)$.
          $endgroup$
          – user479859
          Dec 15 '18 at 9:32














          $begingroup$
          What the means of "$ overline{{f^n(p)|ninmathbb{Z}}}$ has an isolated point"?
          $endgroup$
          – user479859
          Dec 15 '18 at 9:35






          $begingroup$
          What the means of "$ overline{{f^n(p)|ninmathbb{Z}}}$ has an isolated point"?
          $endgroup$
          – user479859
          Dec 15 '18 at 9:35




















          draft saved

          draft discarded




















































          Thanks for contributing an answer to Mathematics Stack Exchange!


          • Please be sure to answer the question. Provide details and share your research!

          But avoid



          • Asking for help, clarification, or responding to other answers.

          • Making statements based on opinion; back them up with references or personal experience.


          Use MathJax to format equations. MathJax reference.


          To learn more, see our tips on writing great answers.




          draft saved


          draft discarded














          StackExchange.ready(
          function () {
          StackExchange.openid.initPostLogin('.new-post-login', 'https%3a%2f%2fmath.stackexchange.com%2fquestions%2f3040299%2fis-this-set-perfect%23new-answer', 'question_page');
          }
          );

          Post as a guest















          Required, but never shown





















































          Required, but never shown














          Required, but never shown












          Required, but never shown







          Required, but never shown

































          Required, but never shown














          Required, but never shown












          Required, but never shown







          Required, but never shown







          Popular posts from this blog

          Plaza Victoria

          In PowerPoint, is there a keyboard shortcut for bulleted / numbered list?

          How to put 3 figures in Latex with 2 figures side by side and 1 below these side by side images but in...